Is abc at least 4?

This topic has expert replies
User avatar
GMAT Instructor
Posts: 1449
Joined: Sat Oct 09, 2010 2:16 pm
Thanked: 59 times
Followed by:33 members

Is abc at least 4?

by fskilnik@GMATH » Fri Feb 08, 2019 6:53 am

Timer

00:00

Your Answer

A

B

C

D

E

Global Stats

GMATH practice exercise (Quant Class 14)

$${\rm{Is}}\,\,abc\,\, \ge \,\,4\,\,\,?$$
$$\left( 1 \right)\,\,b + c \ge 2$$
$$\left( 2 \right)\,\,ab \ge ac \ge 4$$

Answer: [spoiler]____(C)__[/spoiler]
Fabio Skilnik :: GMATH method creator ( Math for the GMAT)
English-speakers :: https://www.gmath.net
Portuguese-speakers :: https://www.gmath.com.br

User avatar
GMAT Instructor
Posts: 15539
Joined: Tue May 25, 2010 12:04 pm
Location: New York, NY
Thanked: 13060 times
Followed by:1906 members
GMAT Score:790

inequalities

by GMATGuruNY » Fri Feb 08, 2019 12:00 pm

Timer

00:00

Your Answer

A

B

C

D

E

Global Stats

fskilnik@GMATH wrote:GMATH practice exercise (Quant Class 14)

$${\rm{Is}}\,\,abc\,\, \ge \,\,4\,\,\,?$$
$$\left( 1 \right)\,\,b + c \ge 2$$
$$\left( 2 \right)\,\,ab \ge ac \ge 4$$
Nice problem, Fabio!

Statement 1:
Case 1: a=4, b=1 and c=1, with the result that b+c≥2
In this case, abc = 4, so the answer to the question stem is YES.
Case 2: a=0, b=1 and c=1, with the result that b+c≥2
In this case, abc = 0, so the answer to the question stem is NO.
Since the answer is YES in Case 1 but NO in Case 2, INSUFFICIENT.

Statement 2:
Case 1: a=4, b=1 and c=1, with the result that ab=4 and ac=4
In this case, abc = 4, so the answer to the question stem is YES.
Case 2: a=-4, b=-1 and c=-1, with the result that ab=4 and ac=4
In this case, abc = -4, so the answer to the question stem is NO.
Since the answer is YES in Case 1 but NO in Case 2, INSUFFICIENT.

Statements combined:
ab ≥ ac ≥ 4 requires that a, b and c have the SAME SIGN.
Since b+c≥2, and b and c have the same sign, b and c must both be POSITIVE.
Implication:
a, b and c are ALL positive.

Thus:
ab ≥ ac
(ab)/a ≥ (ac)/a
b ≥ c

Adding together b ≥ c and b+c ≥ 2, we get:
b + b + c ≥ c + 2
2b ≥ 2
b ≥ 1

Inequalities constrained to positive values can be MULTIPLIED.
Multiplying b≥1 and ac≥4, we get:
abc ≥ 4
Thus, the answer to the question stem is YES.
SUFFICIENT.

The correct answer is C.
Private tutor exclusively for the GMAT and GRE, with over 20 years of experience.
Followed here and elsewhere by over 1900 test-takers.
I have worked with students based in the US, Australia, Taiwan, China, Tajikistan, Kuwait, Saudi Arabia -- a long list of countries.
My students have been admitted to HBS, CBS, Tuck, Yale, Stern, Fuqua -- a long list of top programs.

As a tutor, I don't simply teach you how I would approach problems.
I unlock the best way for YOU to solve problems.

For more information, please email me (Mitch Hunt) at [email protected].
Student Review #1
Student Review #2
Student Review #3

User avatar
GMAT Instructor
Posts: 1449
Joined: Sat Oct 09, 2010 2:16 pm
Thanked: 59 times
Followed by:33 members

Is abc at least 4?

by fskilnik@GMATH » Fri Feb 08, 2019 1:04 pm

Timer

00:00

Your Answer

A

B

C

D

E

Global Stats

fskilnik@GMATH wrote:GMATH practice exercise (Quant Class 14)

$${\rm{Is}}\,\,abc\,\, \ge \,\,4\,\,\,?$$
$$\left( 1 \right)\,\,b + c \ge 2$$
$$\left( 2 \right)\,\,ab \ge ac \ge 4$$
Hi, Mitch! Thanks for the words and for your beautiful contribution!

$$abc\,\,\mathop \ge \limits^? \,\,4$$

$$\left( 1 \right)\,\,b + c\,\, \ge 2\,\,\,\,\,\left\{ \matrix{
\,{\rm{Take}}\,\,\left( {a,b,c} \right) = \left( {1,1,1} \right)\,\,\,\, \Rightarrow \,\,\,\left\langle {{\rm{NO}}} \right\rangle \,\, \hfill \cr
\,{\rm{Take}}\,\,\left( {a,b,c} \right) = \left( {4,1,1} \right)\,\,\,\, \Rightarrow \,\,\,\left\langle {{\rm{YES}}} \right\rangle \,\, \hfill \cr} \right.$$

$$\left( 2 \right)\,\,ab \ge ac \ge 4\,\,\,\,\,\left\{ \matrix{
\,{\rm{(Re)Take}}\,\,\left( {a,b,c} \right) = \left( {4,1,1} \right)\,\,\,\, \Rightarrow \,\,\,\left\langle {{\rm{YES}}} \right\rangle \,\, \hfill \cr
\,{\rm{Take}}\,\,\left( {a,b,c} \right) = \left( { - 2, - 2, - 2} \right)\,\,\,\, \Rightarrow \,\,\,\left\langle {{\rm{NO}}} \right\rangle \,\, \hfill \cr} \right.$$

$$\left( {1 + 2} \right)\,\,a \ne 0\,\,\,\,\,\left( {ac \ne 0} \right)\,\,\,\,::\,\,\,\,\left\{ \matrix{
\,a < 0\,\,\,\, \Rightarrow \,\,\,\,b < 0\,\,\,\,\left( {ab > 0} \right)\,\,\,\,\,and\,\,\,\,\,c < 0\,\,\,\left( {ac > 0} \right)\,\,\,\,\mathop \Rightarrow \limits^{\left( 1 \right)} \,\,\,\,{\rm{impossible}} \hfill \cr
\,a > 0\,\,\,\, \Rightarrow \,\,\,\,b > 0\,\,\,\,\left( {ab > 0} \right)\,\,\,\,\,and\,\,\,\,\,c > 0\,\,\,\left( {ac > 0} \right) \hfill \cr} \right.\,\,\,\,\,\, \Rightarrow \,\,\,\,\,a,b,c\,\,\, > 0\,\,\,\,\,\left( * \right)$$
$$\left. \matrix{
ab \ge 4\,\,\,\,\mathop \Rightarrow \limits^{ \cdot \,\,c\,\,\left( * \right)} \,\,\,abc \ge 4c\,\,\, \hfill \cr
ac \ge 4\,\,\,\,\mathop \Rightarrow \limits^{ \cdot \,\,b\,\,\left( * \right)} \,\,\,abc \ge 4b \hfill \cr} \right\}\,\,\,\,\,\,\,\mathop \Rightarrow \limits^{\left( + \right)} \,\,\,\,\,\,\,2abc \ge 4\left( {b + c} \right)\,\,\,\,\,\,\,\mathop \Rightarrow \limits^{:\,2} \,\,\,\,\,\,\,abc \ge 2\left( {b + c} \right)\,\,\,\,\mathop \Rightarrow \limits^{\left( 1 \right)} \,\,\,\,\left\langle {{\rm{YES}}} \right\rangle $$


The correct answer is therefore (C).


We follow the notations and rationale taught in the GMATH method.

Regards,
Fabio.
Fabio Skilnik :: GMATH method creator ( Math for the GMAT)
English-speakers :: https://www.gmath.net
Portuguese-speakers :: https://www.gmath.com.br